Dadgarnia

New Member
ارسال ها
1,350
لایک ها
1,127
امتیاز
0
#21
پاسخ : ماراتن جبر (سطح ممتاز)




اگه تو معادله قرار بدیم میبینیم صدق نمیکنه!!!​



با مقایسه با رابطه اصلی داریم:


میتونیم راحت هم برای اعداد منفی هم بدست بیاریم!!!
در اول اثباتتون تابع بدست اومده ميشه
كه در رابطه ي اصلي مسئله صدق مي كنه و اونجايي كه مي گيد
اين غلطه با توجه به رابطه ي قبلي كه نوشتيد درستش اينه:
 

math1998

New Member
ارسال ها
336
لایک ها
224
امتیاز
0
#22
پاسخ : ماراتن جبر (سطح ممتاز)

در اول اثباتتون تابع بدست اومده ميشه
كه در رابطه ي اصلي مسئله صدق مي كنه و اونجايي كه مي گيد
اين غلطه با توجه به رابطه ي قبلي كه نوشتيد درستش اينه:
بله حواسم نبود !!!!
 

aras2213

New Member
ارسال ها
216
لایک ها
228
امتیاز
0
#23

Dadgarnia

New Member
ارسال ها
1,350
لایک ها
1,127
امتیاز
0
#24
پاسخ : ماراتن جبر (سطح ممتاز)

سوال بعد:
اگر
و عددی طبیعی باشد تمام توابع
را بیابید به طوریکه داشته باشیم
.
 

math

New Member
ارسال ها
1,129
لایک ها
1,096
امتیاز
0
#25
پاسخ : ماراتن جبر (سطح ممتاز)

سوال بعد:
اگر
و عددی طبیعی باشد تمام توابع
را بیابید به طوریکه داشته باشیم
.
من هم میتونم راه حل بزارم ؟

سوال قشنگی بود واقعا .

یه کلیتی از راه حل . ابتدا x را برابر
قرار دهید (1) و ثابت کنید صفر در برد تابع هستش . حال اسم این عضو رو
بگذارید . به راحتی میتوان دید که توان n ام v هم

ریشه ی f است .( از رابطه ای که از (1) بدست میاد میشه این موضوع رو اثبات کرد .)

حالا خیلی سادس که ببینیم
. حال داریم :


پس معادله تبدیل شد به :




از روابط بالا داشتیم که f تابعی متناوب است و میدانیم که اگر
دوره تناوب باشن تفاضل آن ها نیز دوره تناوب است و استفاده از این نکته که هر چند جمله ی از درجه ی بزرگ تر مساوی 1 از جایی به بعد تمام اعداد رو میپوشونه ثابت کنید چنین توابعی ( غیر ثابت ) وجود ندارد . پس تنها جواب تابع ثابت است .

امید وارم بعد از مدتی فکر نکردن رو مساله جوب نزده باشم .
 

math1998

New Member
ارسال ها
336
لایک ها
224
امتیاز
0
#26
پاسخ : ماراتن جبر (سطح ممتاز)

من هم میتونم راه حل بزارم ؟

سوال قشنگی بود واقعا .

یه کلیتی از راه حل . ابتدا x را برابر
قرار دهید (1) و ثابت کنید صفر در برد تابع هستش . حال اسم این عضو رو
بگذارید . به راحتی میتوان دید که توان n ام v هم

ریشه ی f است .( از رابطه ای که از (1) بدست میاد میشه این موضوع رو اثبات کرد .)

حالا خیلی سادس که ببینیم
. حال داریم :


پس معادله تبدیل شد به :




از روابط بالا داشتیم که f تابعی متناوب است و میدانیم که اگر
دوره تناوب باشن تفاضل آن ها نیز دوره تناوب است و استفاده از این نکته که هر چند جمله ی از درجه ی بزرگ تر مساوی 1 از جایی به بعد تمام اعداد رو میپوشونه ثابت کنید چنین توابعی ( غیر ثابت ) وجود ندارد . پس تنها جواب تابع ثابت است .

امید وارم بعد از مدتی فکر نکردن رو مساله جوب نزده باشم .
راه حل خیلی جالبیه اما من نمیتونم گفته هاتونو به هم بچسبونم مثلا اینکه چطوری ثابت کنیم 0 تو برد هست چرا که در صورت پوشا بودن همچین چیزی درسته یا قسمت اخر راه حلتونو ممکنه بیشتر توضیح بدید؟؟!!
 

math

New Member
ارسال ها
1,129
لایک ها
1,096
امتیاز
0
#27
پاسخ : ماراتن جبر (سطح ممتاز)

راه حل خیلی جالبیه اما من نمیتونم گفته هاتونو به هم بچسبونم مثلا اینکه چطوری ثابت کنیم 0 تو برد هست چرا که در صورت پوشا بودن همچین چیزی درسته یا قسمت اخر راه حلتونو ممکنه بیشتر توضیح بدید؟؟!!

خوب از رابطه ی
.

در باره ی قسمت اخر هم میشه این جوری گفت که برای هر y حقیقی
یک دوره تناوب هستش . فرض کنید یکی از دوره تناوب های تابع t باشد . انگاه داریم :

که
نیز دوره تناوب است . ( توجه کنید که t خودش دوره تناوب است . ) حال میدونیم که تفاضل دو تا دوره تناوب هم یک دوره تناوب هستش . پس داریم

که :
یک دوره تناوب است . پس داریم :
یک دوره تناوب است برای هر y حقیقی اما این یک چند جمله ای از درجه ی n-1 هستش که چون n از دو بیشتره پس درجه این چند جمله ای از 1 بیشتر هستش پس از یک جایی به بعد تمام اعداد رو میپوشونه . پس از یک جایی به بعد تابع ثابت هستش که کاره سختی نیست که ببینید در همه جا ثابته . مگر این که دوره تناوب برابر 0 باشه که نتیجه میده :








اینم سوال بعدی فقط شرمنده من چون ممکنه یه مدتی نیام همراه با سوال لینک جوابش رو هم میذارم ولی اصلا نگاه نکنید چون واقعا سوال قشنگیه حداقل به نظره من :

تمام توابع با دامنه و برد اعداد حقیقی مثبت را بیابید که برای هر دو عضو در دامنه رابطه ی زیر برقرار باشد :








لینک جواب : http://www.artofproblemsolving.com/Forum/viewtopic.php?f=38&t=605662
 

sahands

New Member
ارسال ها
16
لایک ها
1
امتیاز
0
#28
پاسخ : ماراتن جبر (سطح ممتاز)

یک سوال دارم سریع حل کنین
اول تجزیه کنینش چون واسه تعیین علامت میخوتم
 

Dadgarnia

New Member
ارسال ها
1,350
لایک ها
1,127
امتیاز
0
#29
پاسخ : ماراتن جبر (سطح ممتاز)

یک سوال دارم سریع حل کنین
اول تجزیه کنینش چون واسه تعیین علامت میخوتم
فكر نمي كنين اين سوال جاش اينجا نيست؟ خب يه تاپيك توي بخش جبر مقدماتي درست كنين و اونو اونجا بذارين لطفا.
 

sepidfekr

New Member
ارسال ها
711
لایک ها
637
امتیاز
0
#30
پاسخ : ماراتن جبر (سطح ممتاز)

یک سوال دارم سریع حل کنین
اول تجزیه کنینش چون واسه تعیین علامت میخوتم
ببخشیدا ولی خواهشا اعتبار ماراتن رو پایین نیارید اینجا بالاترین سطح سوالات رو میزارن خواهشا تو جبر مقدماتی بزارید
 

Dadgarnia

New Member
ارسال ها
1,350
لایک ها
1,127
امتیاز
0
#31
پاسخ : ماراتن جبر (سطح ممتاز)

سوال بعد:
تمام توابع
را بيابيد كه براي هر
داشته باشيم:

 

aras2213

New Member
ارسال ها
216
لایک ها
228
امتیاز
0
#32
پاسخ : ماراتن جبر (سطح ممتاز)

f ثابت نیست.

1-


---------------------------------------------------------------------------------------------------------------------------------------------------------------

فرض کنید (p(x,y بیان گر معادله باشد.در این صورت
پس اگر (f(0 برابر با 0 نباشد تابع ثابت میشود و این تناقض است.پس
.

حال اگر
، آن گاه خواهیم داشت:
.

2-

---------------------------------------------------------------------------------------------------------------------------------------------------------------


3-f یک به یک است.
---------------------------------------------------------------------------------------------------------------------------------------------------------------
فرض کنید
.داریم:
.

که با استفاده از 2 نتیجه میشود
.

4-f پوشاست
---------------------------------------------------------------------------------------------------------------------------------------------------------------
با استفاده از 2 و رابطه روبرو به راحتی نتیجه میشود f پوشاست:
.



5-
و


---------------------------------------------------------------------------------------------------------------------------------------------------------------
رابطه سمت راست به راحتی با استفاده از 2 و استقرا روی n ثابت میشود.

برای دومی هم اگر در رابطه سمت راست به ترتیب
قرار دهیم ، باتوجه به یک به یک بودن نتیجه میشود:


6-جواب ها :

---------------------------------------------------------------------------------------------------------------------------------------------------------------
با فرض این که برای هر n طبیعی
، بدست می آید:


اگر معادله بدست آمده را (Q(x,y بنامیم، بدست میاد:
.

اگر در این رابطه قرار دهیم y=m بدست میاد:


اگر c=1 نتیجه میگیریم که
.پس با جایگذاری در(Q(x,y و با توجه به پوشا بودن f بدست میاد
.
اگر c=-1 با استفاده از 2 و3و4 تابع فرد میشه و نتیجه میشه
.
حوصله نداشتم چک کنم :4:جوب نداره؟
یه حس خیلی قوی بهم میگه جوب بدی داره:90:
 

Dadgarnia

New Member
ارسال ها
1,350
لایک ها
1,127
امتیاز
0
#33
پاسخ : ماراتن جبر (سطح ممتاز)

f ثابت نیست.

1-


---------------------------------------------------------------------------------------------------------------------------------------------------------------

فرض کنید (p(x,y بیان گر معادله باشد.در این صورت
پس اگر (f(0 برابر با 0 نباشد تابع ثابت میشود و این تناقض است.پس
.

حال اگر
، آن گاه خواهیم داشت:
.

2-

---------------------------------------------------------------------------------------------------------------------------------------------------------------


3-f یک به یک است.
---------------------------------------------------------------------------------------------------------------------------------------------------------------
فرض کنید
.داریم:
.

که با استفاده از 2 نتیجه میشود
.

4-f پوشاست
---------------------------------------------------------------------------------------------------------------------------------------------------------------
با استفاده از 2 و رابطه روبرو به راحتی نتیجه میشود f پوشاست:
.



5-
و


---------------------------------------------------------------------------------------------------------------------------------------------------------------
رابطه سمت راست به راحتی با استفاده از 2 و استقرا روی n ثابت میشود.

برای دومی هم اگر در رابطه سمت راست به ترتیب
قرار دهیم ، باتوجه به یک به یک بودن نتیجه میشود:


6-جواب ها :

---------------------------------------------------------------------------------------------------------------------------------------------------------------
با فرض این که برای هر n طبیعی
، بدست می آید:


اگر معادله بدست آمده را (Q(x,y بنامیم، بدست میاد:
.

اگر در این رابطه قرار دهیم y=m بدست میاد:


اگر c=1 نتیجه میگیریم که
.پس با جایگذاری در(Q(x,y و با توجه به پوشا بودن f بدست میاد
.
اگر c=-1 با استفاده از 2 و3و4 تابع فرد میشه و نتیجه میشه
.
حوصله نداشتم چک کنم :4:جوب نداره؟
یه حس خیلی قوی بهم میگه جوب بدی داره:90:
قسمت 5 رو يكم بيشتر توضيح مي دين؟ اون عبارت كه تك متغيره است! بعدش چرا
؟
 

aras2213

New Member
ارسال ها
216
لایک ها
228
امتیاز
0
#34
پاسخ : ماراتن جبر (سطح ممتاز)

برای قسمت 5 ،


برای دومی هم که گفتید، با توجه به 5،
بعد یه n از طرفین ساده میکنیم.

---- دو نوشته به هم متصل شده است ----

سوال بعد:همه n های طبیعی را بیابید که اعداد حقیقی، دو به دو متمایز و ناصفر
موجود باشند که
.
 

Dadgarnia

New Member
ارسال ها
1,350
لایک ها
1,127
امتیاز
0
#35
پاسخ : ماراتن جبر (سطح ممتاز)

به علت خوابیدن ماراتن من سوال بعدو میذارم هر کی قبلی رو حل کرد جوابشو بنویسه.
سوال بعد:
همه ی چند جمله ای های
با ضرایب حقیقی را بیابید به طوریکه برای هر سه عدد حقیقی
داشته باشیم:


 

AHZolfaghari

Well-Known Member
ارسال ها
935
لایک ها
1,654
امتیاز
93
#36
پاسخ : ماراتن جبر (سطح ممتاز)

به علت خوابیدن ماراتن من سوال بعدو میذارم هر کی قبلی رو حل کرد جوابشو بنویسه.
سوال بعد:
همه ی چند جمله ای های
با ضرایب حقیقی را بیابید به طوریکه برای هر سه عدد حقیقی
داشته باشیم:


B,C رو صفر میدیم :




ضریب پیشرو چندجمله ای رو هم میگیریم
چندجمله ای رو از درجه n میگیریم

حالا ضریب
رو در دو طرف حساب میکنیم و برابر قرار میدیم .



حالا اگه n فرد باشه طرف چپ صفر میشه و خواهیم داشت n = 1

که با بررسی میتوان یافت که صدق میکنند

اگه n زوج باشه


که اینم تناقض داره چون یه طرف زوج و طرف دیگه فرد میشه ( اگه n بزرگ تر مساوی دو باشه )

پس فقط چندجمله ای ثابت و کلیه چندجمله ای های درجه اول صدق میکنند

درسته راه حلم ؟​
 
آخرین ویرایش توسط مدیر

Dadgarnia

New Member
ارسال ها
1,350
لایک ها
1,127
امتیاز
0
#37
پاسخ : ماراتن جبر (سطح ممتاز)

B,C رو صفر میدیم :




ضریب پیشرو چندجمله ای رو هم میگیریم
چندجمله ای رو از درجه n میگیریم

حالا ضریب
رو در دو طرف حساب میکنیم و برابر قرار میدیم .



حالا اگه n فرد باشه طرف چپ صفر میشه و خواهیم داشت n = 1

که با بررسی میتوان یافت که صدق میکنند

اگه n زوج باشه


که اینم تناقض داره چون یه طرف زوج و طرف دیگه فرد میشه ( اگه n بزرگ تر مساوی دو باشه )

پس فقط چندجمله ای ثابت و کلیه چندجمله ای های درجه اول صدق میکنند

درسته راه حلم ؟​
فقط توي حالت دوم n=2 صدق مي كنه كه بعدش بايد بررسي بشه. لطفا سوال بعد رو هم بذاريد.
 

AHZolfaghari

Well-Known Member
ارسال ها
935
لایک ها
1,654
امتیاز
93
#38
پاسخ : ماراتن جبر (سطح ممتاز)

فقط توي حالت دوم n=2 صدق مي كنه كه بعدش بايد بررسي بشه. لطفا سوال بعد رو هم بذاريد.
بله درسته من به اشتباه نوشتم 1 که باید مینوشتم 2 که حالت N=2 رو در پی داره .

البته زیاد سخت نیست

اگه



مینیموم



رو بیابید​
 

sepidfekr

New Member
ارسال ها
711
لایک ها
637
امتیاز
0
#39
پاسخ : ماراتن جبر (سطح ممتاز)

بله درسته من به اشتباه نوشتم 1 که باید مینوشتم 2 که حالت N=2 رو در پی داره .

البته زیاد سخت نیست

اگه



مینیموم



رو بیابید​
مینیمم 2 میشه؟
 

AHZolfaghari

Well-Known Member
ارسال ها
935
لایک ها
1,654
امتیاز
93
#40
پاسخ : ماراتن جبر (سطح ممتاز)


با راه حل صحبت کن !!!​

---- دو نوشته به هم متصل شده است ----

البته بگم که x,y,z,t نا منفی هستند . یادم رفته بود بنویسم [MENTION=19999]sepidfekr[/MENTION]
 
بالا